Trying to parse a Putnam solution from 1995

The name of the pictureThe name of the pictureThe name of the pictureClash Royale CLAN TAG#URR8PPP












5












$begingroup$


I’m having trouble parsing the solution for the 1995 Putnam, question A2. The proof proceeds:



The easiest proof uses
``big-O'' notation and the fact that $(1+x)^1/2 = 1 + x/2 +
O(x^2)$
for $|x|<1$. (Here $O(x^2)$ means bounded by a constant
times $x^2$.) Me: That’s all well and good, and I can arrive at that by the generalized binomial theorem.



The proof continues:
beginalign*
sqrtx+a-sqrtx &= x^1/2(sqrt1+a/x - 1) \
&= x^1/2(1 + a/2x + O(x^-2)),
endalign*



It would appear that’s an error; it would appear the proof writer dropped the minus 1.



But it continues, hence



$sqrtsqrtx+a - sqrtx = x^1/4 (a/4x + O(x^-2))$



Which I’m at a loss how to arrive at.










share|cite|improve this question











$endgroup$











  • $begingroup$
    For what it’s worth, you aren’t the first person to have asked about this. math.stackexchange.com/questions/2020837/…
    $endgroup$
    – Steve Kass
    Dec 30 '18 at 0:44










  • $begingroup$
    I am not sure if you have copied things properly. Why do we have $O(x^2)$ in one place and $O(x^-2)$ at others?
    $endgroup$
    – Kavi Rama Murthy
    Dec 30 '18 at 0:45










  • $begingroup$
    Note that on the right side of the first equation, x is in the denominator, whereas at the top, x is in the numerator.
    $endgroup$
    – Thor Kamphefner
    Dec 30 '18 at 0:46










  • $begingroup$
    @KaviRamaMurthy Not OP, but that is because the new "$x$" is now $a/x$, so when you substitute that in $O(x^2)$ becomes $O(a^2/x^2)$, which is the same as $O(x^-2)$
    $endgroup$
    – Noble Mushtak
    Dec 30 '18 at 0:46










  • $begingroup$
    You should show the question. I looked up the reference and it doesn't appear to be what you want. Also, where is the proof that you refer to?
    $endgroup$
    – herb steinberg
    Dec 30 '18 at 0:46















5












$begingroup$


I’m having trouble parsing the solution for the 1995 Putnam, question A2. The proof proceeds:



The easiest proof uses
``big-O'' notation and the fact that $(1+x)^1/2 = 1 + x/2 +
O(x^2)$
for $|x|<1$. (Here $O(x^2)$ means bounded by a constant
times $x^2$.) Me: That’s all well and good, and I can arrive at that by the generalized binomial theorem.



The proof continues:
beginalign*
sqrtx+a-sqrtx &= x^1/2(sqrt1+a/x - 1) \
&= x^1/2(1 + a/2x + O(x^-2)),
endalign*



It would appear that’s an error; it would appear the proof writer dropped the minus 1.



But it continues, hence



$sqrtsqrtx+a - sqrtx = x^1/4 (a/4x + O(x^-2))$



Which I’m at a loss how to arrive at.










share|cite|improve this question











$endgroup$











  • $begingroup$
    For what it’s worth, you aren’t the first person to have asked about this. math.stackexchange.com/questions/2020837/…
    $endgroup$
    – Steve Kass
    Dec 30 '18 at 0:44










  • $begingroup$
    I am not sure if you have copied things properly. Why do we have $O(x^2)$ in one place and $O(x^-2)$ at others?
    $endgroup$
    – Kavi Rama Murthy
    Dec 30 '18 at 0:45










  • $begingroup$
    Note that on the right side of the first equation, x is in the denominator, whereas at the top, x is in the numerator.
    $endgroup$
    – Thor Kamphefner
    Dec 30 '18 at 0:46










  • $begingroup$
    @KaviRamaMurthy Not OP, but that is because the new "$x$" is now $a/x$, so when you substitute that in $O(x^2)$ becomes $O(a^2/x^2)$, which is the same as $O(x^-2)$
    $endgroup$
    – Noble Mushtak
    Dec 30 '18 at 0:46










  • $begingroup$
    You should show the question. I looked up the reference and it doesn't appear to be what you want. Also, where is the proof that you refer to?
    $endgroup$
    – herb steinberg
    Dec 30 '18 at 0:46













5












5








5





$begingroup$


I’m having trouble parsing the solution for the 1995 Putnam, question A2. The proof proceeds:



The easiest proof uses
``big-O'' notation and the fact that $(1+x)^1/2 = 1 + x/2 +
O(x^2)$
for $|x|<1$. (Here $O(x^2)$ means bounded by a constant
times $x^2$.) Me: That’s all well and good, and I can arrive at that by the generalized binomial theorem.



The proof continues:
beginalign*
sqrtx+a-sqrtx &= x^1/2(sqrt1+a/x - 1) \
&= x^1/2(1 + a/2x + O(x^-2)),
endalign*



It would appear that’s an error; it would appear the proof writer dropped the minus 1.



But it continues, hence



$sqrtsqrtx+a - sqrtx = x^1/4 (a/4x + O(x^-2))$



Which I’m at a loss how to arrive at.










share|cite|improve this question











$endgroup$




I’m having trouble parsing the solution for the 1995 Putnam, question A2. The proof proceeds:



The easiest proof uses
``big-O'' notation and the fact that $(1+x)^1/2 = 1 + x/2 +
O(x^2)$
for $|x|<1$. (Here $O(x^2)$ means bounded by a constant
times $x^2$.) Me: That’s all well and good, and I can arrive at that by the generalized binomial theorem.



The proof continues:
beginalign*
sqrtx+a-sqrtx &= x^1/2(sqrt1+a/x - 1) \
&= x^1/2(1 + a/2x + O(x^-2)),
endalign*



It would appear that’s an error; it would appear the proof writer dropped the minus 1.



But it continues, hence



$sqrtsqrtx+a - sqrtx = x^1/4 (a/4x + O(x^-2))$



Which I’m at a loss how to arrive at.







binomial-theorem






share|cite|improve this question















share|cite|improve this question













share|cite|improve this question




share|cite|improve this question








edited Dec 30 '18 at 0:47







Thor Kamphefner

















asked Dec 30 '18 at 0:36









Thor KamphefnerThor Kamphefner

445




445











  • $begingroup$
    For what it’s worth, you aren’t the first person to have asked about this. math.stackexchange.com/questions/2020837/…
    $endgroup$
    – Steve Kass
    Dec 30 '18 at 0:44










  • $begingroup$
    I am not sure if you have copied things properly. Why do we have $O(x^2)$ in one place and $O(x^-2)$ at others?
    $endgroup$
    – Kavi Rama Murthy
    Dec 30 '18 at 0:45










  • $begingroup$
    Note that on the right side of the first equation, x is in the denominator, whereas at the top, x is in the numerator.
    $endgroup$
    – Thor Kamphefner
    Dec 30 '18 at 0:46










  • $begingroup$
    @KaviRamaMurthy Not OP, but that is because the new "$x$" is now $a/x$, so when you substitute that in $O(x^2)$ becomes $O(a^2/x^2)$, which is the same as $O(x^-2)$
    $endgroup$
    – Noble Mushtak
    Dec 30 '18 at 0:46










  • $begingroup$
    You should show the question. I looked up the reference and it doesn't appear to be what you want. Also, where is the proof that you refer to?
    $endgroup$
    – herb steinberg
    Dec 30 '18 at 0:46
















  • $begingroup$
    For what it’s worth, you aren’t the first person to have asked about this. math.stackexchange.com/questions/2020837/…
    $endgroup$
    – Steve Kass
    Dec 30 '18 at 0:44










  • $begingroup$
    I am not sure if you have copied things properly. Why do we have $O(x^2)$ in one place and $O(x^-2)$ at others?
    $endgroup$
    – Kavi Rama Murthy
    Dec 30 '18 at 0:45










  • $begingroup$
    Note that on the right side of the first equation, x is in the denominator, whereas at the top, x is in the numerator.
    $endgroup$
    – Thor Kamphefner
    Dec 30 '18 at 0:46










  • $begingroup$
    @KaviRamaMurthy Not OP, but that is because the new "$x$" is now $a/x$, so when you substitute that in $O(x^2)$ becomes $O(a^2/x^2)$, which is the same as $O(x^-2)$
    $endgroup$
    – Noble Mushtak
    Dec 30 '18 at 0:46










  • $begingroup$
    You should show the question. I looked up the reference and it doesn't appear to be what you want. Also, where is the proof that you refer to?
    $endgroup$
    – herb steinberg
    Dec 30 '18 at 0:46















$begingroup$
For what it’s worth, you aren’t the first person to have asked about this. math.stackexchange.com/questions/2020837/…
$endgroup$
– Steve Kass
Dec 30 '18 at 0:44




$begingroup$
For what it’s worth, you aren’t the first person to have asked about this. math.stackexchange.com/questions/2020837/…
$endgroup$
– Steve Kass
Dec 30 '18 at 0:44












$begingroup$
I am not sure if you have copied things properly. Why do we have $O(x^2)$ in one place and $O(x^-2)$ at others?
$endgroup$
– Kavi Rama Murthy
Dec 30 '18 at 0:45




$begingroup$
I am not sure if you have copied things properly. Why do we have $O(x^2)$ in one place and $O(x^-2)$ at others?
$endgroup$
– Kavi Rama Murthy
Dec 30 '18 at 0:45












$begingroup$
Note that on the right side of the first equation, x is in the denominator, whereas at the top, x is in the numerator.
$endgroup$
– Thor Kamphefner
Dec 30 '18 at 0:46




$begingroup$
Note that on the right side of the first equation, x is in the denominator, whereas at the top, x is in the numerator.
$endgroup$
– Thor Kamphefner
Dec 30 '18 at 0:46












$begingroup$
@KaviRamaMurthy Not OP, but that is because the new "$x$" is now $a/x$, so when you substitute that in $O(x^2)$ becomes $O(a^2/x^2)$, which is the same as $O(x^-2)$
$endgroup$
– Noble Mushtak
Dec 30 '18 at 0:46




$begingroup$
@KaviRamaMurthy Not OP, but that is because the new "$x$" is now $a/x$, so when you substitute that in $O(x^2)$ becomes $O(a^2/x^2)$, which is the same as $O(x^-2)$
$endgroup$
– Noble Mushtak
Dec 30 '18 at 0:46












$begingroup$
You should show the question. I looked up the reference and it doesn't appear to be what you want. Also, where is the proof that you refer to?
$endgroup$
– herb steinberg
Dec 30 '18 at 0:46




$begingroup$
You should show the question. I looked up the reference and it doesn't appear to be what you want. Also, where is the proof that you refer to?
$endgroup$
– herb steinberg
Dec 30 '18 at 0:46










1 Answer
1






active

oldest

votes


















9












$begingroup$

OK, so the first step is clearly a mistake with them dropping the $-1$. However, I'm going to try to explain what they may have done, even though it doesn't make sense. First, let's start with their (albeit wrong) equation:



$$sqrtx+a-sqrtx=x^1/2(1+fraca2x+O(x^-2))$$
Now, take the square root:
$$sqrtsqrtx+a-sqrtx=[x^1/2(1+fraca2x+O(x^-2))]^1/2$$



Distribute the exponent:
$$sqrtsqrtx+a-sqrtx=x^1/4[(1+fraca2x+O(x^-2))]^1/2$$



Now, the second expression on the right is in the form of $(1+z)^1/2$, so I will use the following formula:



$$(1+z)^1/2=1+frac z 2+O(z^2)rightarrow \ (1+frac a2x+O(x^-2))^1/2=1+fraca4x+frac12O(x^-2)+O(fraca^24x^2)=1+fraca4x+O(x^-2)$$



Thus, we get:



$$sqrtsqrtx+a-sqrtx=x^1/4(1+fraca4x+O(x^-2))$$



Oh, and remember that $-1$ we dropped in the first step? Let's try adding that back in here, because...why not?



$$sqrtsqrtx+a-sqrtx=x^1/4(1+fraca4x+O(x^-2)-1)$$
$$sqrtsqrtx+a-sqrtx=x^1/4(fraca4x+O(x^-2))$$
Thus, we get the same equation they did, and we only had to make two mathematical errors to get there!






share|cite|improve this answer









$endgroup$












  • $begingroup$
    I like that you were able to decipher what the original proof writer most likely meant. Thank you for that.
    $endgroup$
    – Thor Kamphefner
    Dec 30 '18 at 0:56










Your Answer





StackExchange.ifUsing("editor", function ()
return StackExchange.using("mathjaxEditing", function ()
StackExchange.MarkdownEditor.creationCallbacks.add(function (editor, postfix)
StackExchange.mathjaxEditing.prepareWmdForMathJax(editor, postfix, [["$", "$"], ["\\(","\\)"]]);
);
);
, "mathjax-editing");

StackExchange.ready(function()
var channelOptions =
tags: "".split(" "),
id: "69"
;
initTagRenderer("".split(" "), "".split(" "), channelOptions);

StackExchange.using("externalEditor", function()
// Have to fire editor after snippets, if snippets enabled
if (StackExchange.settings.snippets.snippetsEnabled)
StackExchange.using("snippets", function()
createEditor();
);

else
createEditor();

);

function createEditor()
StackExchange.prepareEditor(
heartbeatType: 'answer',
autoActivateHeartbeat: false,
convertImagesToLinks: true,
noModals: true,
showLowRepImageUploadWarning: true,
reputationToPostImages: 10,
bindNavPrevention: true,
postfix: "",
imageUploader:
brandingHtml: "Powered by u003ca class="icon-imgur-white" href="https://imgur.com/"u003eu003c/au003e",
contentPolicyHtml: "User contributions licensed under u003ca href="https://creativecommons.org/licenses/by-sa/3.0/"u003ecc by-sa 3.0 with attribution requiredu003c/au003e u003ca href="https://stackoverflow.com/legal/content-policy"u003e(content policy)u003c/au003e",
allowUrls: true
,
noCode: true, onDemand: true,
discardSelector: ".discard-answer"
,immediatelyShowMarkdownHelp:true
);



);













draft saved

draft discarded


















StackExchange.ready(
function ()
StackExchange.openid.initPostLogin('.new-post-login', 'https%3a%2f%2fmath.stackexchange.com%2fquestions%2f3056405%2ftrying-to-parse-a-putnam-solution-from-1995%23new-answer', 'question_page');

);

Post as a guest















Required, but never shown

























1 Answer
1






active

oldest

votes








1 Answer
1






active

oldest

votes









active

oldest

votes






active

oldest

votes









9












$begingroup$

OK, so the first step is clearly a mistake with them dropping the $-1$. However, I'm going to try to explain what they may have done, even though it doesn't make sense. First, let's start with their (albeit wrong) equation:



$$sqrtx+a-sqrtx=x^1/2(1+fraca2x+O(x^-2))$$
Now, take the square root:
$$sqrtsqrtx+a-sqrtx=[x^1/2(1+fraca2x+O(x^-2))]^1/2$$



Distribute the exponent:
$$sqrtsqrtx+a-sqrtx=x^1/4[(1+fraca2x+O(x^-2))]^1/2$$



Now, the second expression on the right is in the form of $(1+z)^1/2$, so I will use the following formula:



$$(1+z)^1/2=1+frac z 2+O(z^2)rightarrow \ (1+frac a2x+O(x^-2))^1/2=1+fraca4x+frac12O(x^-2)+O(fraca^24x^2)=1+fraca4x+O(x^-2)$$



Thus, we get:



$$sqrtsqrtx+a-sqrtx=x^1/4(1+fraca4x+O(x^-2))$$



Oh, and remember that $-1$ we dropped in the first step? Let's try adding that back in here, because...why not?



$$sqrtsqrtx+a-sqrtx=x^1/4(1+fraca4x+O(x^-2)-1)$$
$$sqrtsqrtx+a-sqrtx=x^1/4(fraca4x+O(x^-2))$$
Thus, we get the same equation they did, and we only had to make two mathematical errors to get there!






share|cite|improve this answer









$endgroup$












  • $begingroup$
    I like that you were able to decipher what the original proof writer most likely meant. Thank you for that.
    $endgroup$
    – Thor Kamphefner
    Dec 30 '18 at 0:56















9












$begingroup$

OK, so the first step is clearly a mistake with them dropping the $-1$. However, I'm going to try to explain what they may have done, even though it doesn't make sense. First, let's start with their (albeit wrong) equation:



$$sqrtx+a-sqrtx=x^1/2(1+fraca2x+O(x^-2))$$
Now, take the square root:
$$sqrtsqrtx+a-sqrtx=[x^1/2(1+fraca2x+O(x^-2))]^1/2$$



Distribute the exponent:
$$sqrtsqrtx+a-sqrtx=x^1/4[(1+fraca2x+O(x^-2))]^1/2$$



Now, the second expression on the right is in the form of $(1+z)^1/2$, so I will use the following formula:



$$(1+z)^1/2=1+frac z 2+O(z^2)rightarrow \ (1+frac a2x+O(x^-2))^1/2=1+fraca4x+frac12O(x^-2)+O(fraca^24x^2)=1+fraca4x+O(x^-2)$$



Thus, we get:



$$sqrtsqrtx+a-sqrtx=x^1/4(1+fraca4x+O(x^-2))$$



Oh, and remember that $-1$ we dropped in the first step? Let's try adding that back in here, because...why not?



$$sqrtsqrtx+a-sqrtx=x^1/4(1+fraca4x+O(x^-2)-1)$$
$$sqrtsqrtx+a-sqrtx=x^1/4(fraca4x+O(x^-2))$$
Thus, we get the same equation they did, and we only had to make two mathematical errors to get there!






share|cite|improve this answer









$endgroup$












  • $begingroup$
    I like that you were able to decipher what the original proof writer most likely meant. Thank you for that.
    $endgroup$
    – Thor Kamphefner
    Dec 30 '18 at 0:56













9












9








9





$begingroup$

OK, so the first step is clearly a mistake with them dropping the $-1$. However, I'm going to try to explain what they may have done, even though it doesn't make sense. First, let's start with their (albeit wrong) equation:



$$sqrtx+a-sqrtx=x^1/2(1+fraca2x+O(x^-2))$$
Now, take the square root:
$$sqrtsqrtx+a-sqrtx=[x^1/2(1+fraca2x+O(x^-2))]^1/2$$



Distribute the exponent:
$$sqrtsqrtx+a-sqrtx=x^1/4[(1+fraca2x+O(x^-2))]^1/2$$



Now, the second expression on the right is in the form of $(1+z)^1/2$, so I will use the following formula:



$$(1+z)^1/2=1+frac z 2+O(z^2)rightarrow \ (1+frac a2x+O(x^-2))^1/2=1+fraca4x+frac12O(x^-2)+O(fraca^24x^2)=1+fraca4x+O(x^-2)$$



Thus, we get:



$$sqrtsqrtx+a-sqrtx=x^1/4(1+fraca4x+O(x^-2))$$



Oh, and remember that $-1$ we dropped in the first step? Let's try adding that back in here, because...why not?



$$sqrtsqrtx+a-sqrtx=x^1/4(1+fraca4x+O(x^-2)-1)$$
$$sqrtsqrtx+a-sqrtx=x^1/4(fraca4x+O(x^-2))$$
Thus, we get the same equation they did, and we only had to make two mathematical errors to get there!






share|cite|improve this answer









$endgroup$



OK, so the first step is clearly a mistake with them dropping the $-1$. However, I'm going to try to explain what they may have done, even though it doesn't make sense. First, let's start with their (albeit wrong) equation:



$$sqrtx+a-sqrtx=x^1/2(1+fraca2x+O(x^-2))$$
Now, take the square root:
$$sqrtsqrtx+a-sqrtx=[x^1/2(1+fraca2x+O(x^-2))]^1/2$$



Distribute the exponent:
$$sqrtsqrtx+a-sqrtx=x^1/4[(1+fraca2x+O(x^-2))]^1/2$$



Now, the second expression on the right is in the form of $(1+z)^1/2$, so I will use the following formula:



$$(1+z)^1/2=1+frac z 2+O(z^2)rightarrow \ (1+frac a2x+O(x^-2))^1/2=1+fraca4x+frac12O(x^-2)+O(fraca^24x^2)=1+fraca4x+O(x^-2)$$



Thus, we get:



$$sqrtsqrtx+a-sqrtx=x^1/4(1+fraca4x+O(x^-2))$$



Oh, and remember that $-1$ we dropped in the first step? Let's try adding that back in here, because...why not?



$$sqrtsqrtx+a-sqrtx=x^1/4(1+fraca4x+O(x^-2)-1)$$
$$sqrtsqrtx+a-sqrtx=x^1/4(fraca4x+O(x^-2))$$
Thus, we get the same equation they did, and we only had to make two mathematical errors to get there!







share|cite|improve this answer












share|cite|improve this answer



share|cite|improve this answer










answered Dec 30 '18 at 0:52









Noble MushtakNoble Mushtak

15.2k1735




15.2k1735











  • $begingroup$
    I like that you were able to decipher what the original proof writer most likely meant. Thank you for that.
    $endgroup$
    – Thor Kamphefner
    Dec 30 '18 at 0:56
















  • $begingroup$
    I like that you were able to decipher what the original proof writer most likely meant. Thank you for that.
    $endgroup$
    – Thor Kamphefner
    Dec 30 '18 at 0:56















$begingroup$
I like that you were able to decipher what the original proof writer most likely meant. Thank you for that.
$endgroup$
– Thor Kamphefner
Dec 30 '18 at 0:56




$begingroup$
I like that you were able to decipher what the original proof writer most likely meant. Thank you for that.
$endgroup$
– Thor Kamphefner
Dec 30 '18 at 0:56

















draft saved

draft discarded
















































Thanks for contributing an answer to Mathematics Stack Exchange!


  • Please be sure to answer the question. Provide details and share your research!

But avoid


  • Asking for help, clarification, or responding to other answers.

  • Making statements based on opinion; back them up with references or personal experience.

Use MathJax to format equations. MathJax reference.


To learn more, see our tips on writing great answers.




draft saved


draft discarded














StackExchange.ready(
function ()
StackExchange.openid.initPostLogin('.new-post-login', 'https%3a%2f%2fmath.stackexchange.com%2fquestions%2f3056405%2ftrying-to-parse-a-putnam-solution-from-1995%23new-answer', 'question_page');

);

Post as a guest















Required, but never shown





















































Required, but never shown














Required, but never shown












Required, but never shown







Required, but never shown

































Required, but never shown














Required, but never shown












Required, but never shown







Required, but never shown






Popular posts from this blog

How to check contact read email or not when send email to Individual?

Bahrain

Postfix configuration issue with fips on centos 7; mailgun relay